Saltar al contenido

Desigualdad con dos valores absolutos

Solución:

Cómo resolver $ | x-3 | – | x-4 |

Comenzaría señalando $ | x-3 | = x-3 $ para $ x geq 3 $ mientras que $ | x-3 | = 3-x $ para $ x leq 3 $. Del mismo modo, $ | x-4 | = x-4 $ para $ x geq 4 $ mientras que $ | x-4 | = 4-x $ para $ x leq 4 $

  1. si $ x geq 4 $ entonces $ x> 3 $ por tanto $ f (x) = x-3- (x-4) = 1 $ y encontramos $ 1[4,infty)$ in the solution set of the inequality.

  2. if $x leq 3$ then $x < 4$ hence $f(x)=3-x-(4-x)=-1$ and we face $-1<x$ which is true for each $x$ with $-1<xleq 3$. This shows $(-1,3]$ también está en el conjunto solución de la desigualdad.

  3. si $ 3

Ponga todo esto junto, obtenemos $ (- 1,3] cup (3,4) cup[4infty)=(-1infty)$queessurespuestaPorlogeneralresuelvoestetipodecosasconunmétododegráficodesignos[4infty)=(-1infty)$whichisyouranswerUsuallyIsolvethissortofthingwithasign-chartmethod

La forma semimecánica estándar de eliminar los signos de valor absoluto es dividir la recta numérica en segmentos. El punto crítico para $ | x-4 | $ está en $ x = 4 $, y el punto crítico para $ | x-3 | $ está en $ x = 3 $.

Suponga primero que $ x ge 4 $. Entonces $ | x-4 | = x-4 $ y $ | x-3 | = x-3 $. Entonces, estamos viendo la desigualdad $ (x-3) – (x-4) lt x $, es decir, en $ 1 lt x $, lo que ciertamente es cierto para $ x ge 4 $.

Ahora suponga que $ 3 le x lt 4 $. Entonces $ | x-4 | = 4-x $ y $ | x = 3 | = x-3 $, entonces estamos viendo la desigualdad $ (x-3) – (4-x) lt x $, que es, $ 2x-7 lt x $. Esto se simplifica a $ x lt 7 $, que ciertamente es cierto en el intervalo $[34)$[34)$

Probablemente sea en este punto cuando su cálculo se descarrió. Observamos el intervalo $[34)$ypreguntamosquépuntos[34)$andaskingwhichpointsen este intervalo satisfecho nuestra desigualdad. La manipulación nos dijo que eran todos los puntos de este intervalo los que satisfacían $ x lt 7 $. Bueno, ¡todos lo hacen!

Finalmente, suponga que $ x lt 3 $. Entonces $ | x-4 | = 4-x $ y $ | x-3 | = 3-x $. Entonces, estamos viendo la desigualdad $ (3-x) – (4-x) lt x $. Calcular. El lado izquierdo es $ -1 $, por lo que en el intervalo $ (- infty, 3) $, la desigualdad se mantiene precisamente cuando $ -1 lt x $.

Poniendo las cosas juntas, llegamos a la conclusión de que la desigualdad original se cumple (i) si $ x ge 4 $; (ii) si $ 3 le x lt 4 $; y (ii) si $ x lt 3 $ pero $ -1 lt x $. Este complicado conjunto de condiciones se puede resumir mucho más simplemente como $ x gt -1 $.

Hay otras formas de describir el conjunto de soluciones. Por ejemplo, podríamos decir que el conjunto es $ (- 1, infty) $.

Lo que tienes es casi correcto, el último paso es restringir tu solución a la región correspondiente.

Por ejemplo, para $ x> 1 $, la respuesta debe estar en el $ x> 4 $ región, por lo que su respuesta para esta región es $ x> 4 $.

Para $ x> -1 $, tu respuesta debe estar en $ x <3 $ región, por lo que su respuesta para esta región es $ -1.

Y para el ultimo $ x <7 $, su respuesta debe estar en la región correspondiente que teníamos primero, es decir $ 3, entonces tu respuesta para esta región es $ 3.

Ahora dibuja estas tres respuestas en la recta numérica y tendrás $ x> -1 $, la respuesta final deseada.

¡Haz clic para puntuar esta entrada!
(Votos: 0 Promedio: 0)



Utiliza Nuestro Buscador

Deja una respuesta

Tu dirección de correo electrónico no será publicada. Los campos obligatorios están marcados con *